Find the sum of series: $sum_{k=0}^{n} (-1)^k binom{n}{k} binom{2n-k}{n}$












2












$begingroup$


To find the sum:
$$sum_{k=0}^{n} (-1)^k binom{n}{k} binom{2n-k}{n}$$



Try:



I do not have any clue about the question. I was thinking of finding coefficient of some required power in a binomial expansion, but wasn't able to proceed as the power of $x$ seems to be non-constant in each term ($x^{n+k}$).



Please give a small hint!










share|cite|improve this question











$endgroup$

















    2












    $begingroup$


    To find the sum:
    $$sum_{k=0}^{n} (-1)^k binom{n}{k} binom{2n-k}{n}$$



    Try:



    I do not have any clue about the question. I was thinking of finding coefficient of some required power in a binomial expansion, but wasn't able to proceed as the power of $x$ seems to be non-constant in each term ($x^{n+k}$).



    Please give a small hint!










    share|cite|improve this question











    $endgroup$















      2












      2








      2


      2



      $begingroup$


      To find the sum:
      $$sum_{k=0}^{n} (-1)^k binom{n}{k} binom{2n-k}{n}$$



      Try:



      I do not have any clue about the question. I was thinking of finding coefficient of some required power in a binomial expansion, but wasn't able to proceed as the power of $x$ seems to be non-constant in each term ($x^{n+k}$).



      Please give a small hint!










      share|cite|improve this question











      $endgroup$




      To find the sum:
      $$sum_{k=0}^{n} (-1)^k binom{n}{k} binom{2n-k}{n}$$



      Try:



      I do not have any clue about the question. I was thinking of finding coefficient of some required power in a binomial expansion, but wasn't able to proceed as the power of $x$ seems to be non-constant in each term ($x^{n+k}$).



      Please give a small hint!







      summation binomial-coefficients binomial-theorem






      share|cite|improve this question















      share|cite|improve this question













      share|cite|improve this question




      share|cite|improve this question








      edited Jun 25 '17 at 14:51









      uniquesolution

      9,1711823




      9,1711823










      asked Jun 25 '17 at 14:49









      samjoesamjoe

      6,13311028




      6,13311028






















          5 Answers
          5






          active

          oldest

          votes


















          2












          $begingroup$

          $newcommand{bbx}[1]{,bbox[15px,border:1px groove navy]{displaystyle{#1}},}
          newcommand{braces}[1]{leftlbrace,{#1},rightrbrace}
          newcommand{bracks}[1]{leftlbrack,{#1},rightrbrack}
          newcommand{dd}{mathrm{d}}
          newcommand{ds}[1]{displaystyle{#1}}
          newcommand{expo}[1]{,mathrm{e}^{#1},}
          newcommand{ic}{mathrm{i}}
          newcommand{mc}[1]{mathcal{#1}}
          newcommand{mrm}[1]{mathrm{#1}}
          newcommand{pars}[1]{left(,{#1},right)}
          newcommand{partiald}[3]{frac{partial^{#1} #2}{partial #3^{#1}}}
          newcommand{root}[2]{,sqrt[#1]{,{#2},},}
          newcommand{totald}[3]{frac{mathrm{d}^{#1} #2}{mathrm{d} #3^{#1}}}
          newcommand{verts}[1]{leftvert,{#1},rightvert}$
          begin{align}
          sum_{k = 0}^{n}pars{-1}^{k}{n choose k}{2n - k choose n} & =
          sum_{k = 0}^{n}pars{-1}^{k}{n choose k}bracks{z^{n}}pars{1 + z}^{2n - k}
          \[5mm] & =
          bracks{z^{n}}pars{1 + z}^{2n}
          sum_{k = 0}^{n}{n choose k}pars{-,{1 over 1 + z}}^{k}
          \[5mm] & =
          bracks{z^{n}}pars{1 + z}^{2n}pars{1 - {1 over 1 + z}}^{n} =
          bracks{z^{n}}pars{1 + z}^{n},z^{n} = bbx{large 1}
          end{align}






          share|cite|improve this answer









          $endgroup$













          • $begingroup$
            Thanks for your answer! I think I need to practice more on algebraic manipulation.
            $endgroup$
            – samjoe
            Jun 26 '17 at 3:31










          • $begingroup$
            That's a good idea. Thanks. $left(angle quad angle atop {Huge smile}right)$
            $endgroup$
            – Felix Marin
            Jun 26 '17 at 3:34










          • $begingroup$
            Sorry for off-topic question - do you type out latex or use some software?
            $endgroup$
            – samjoe
            Jun 26 '17 at 3:45










          • $begingroup$
            I type $LaTeX$. The above icon is $LaTeX$ too. It's the code: $texttt{$left(anglequadangleatop{Hugesmile}right)$}$
            $endgroup$
            – Felix Marin
            Jun 26 '17 at 3:59





















          2












          $begingroup$

          Note that
          begin{align*}
          sum_{k=0}^{n} (-1)^k binom{n}{k} binom{2n-k}{n}&=sum_{k=0}^{n} (-1)^k binom{n}{k} binom{2n-k}{n-k}\
          &=sum_{k=0}^{n} (-1)^k binom{n}{k} (-1)^{n-k}binom{-(n+1)}{n-k}\
          &=(-1)^{n}sum_{k=0}^{n} binom{n}{k} binom{-(n+1)}{n-k}\
          &=(-1)^{n}binom{n-(n+1)}{n}=1.
          end{align*}
          where we used
          $$binom{2n-k}{n-k}=frac{(2n-k)cdots(n+1)}{(n-k)!}=
          (-1)^{n-k}frac{(-n-1)cdots(-2n+k)}{(n-k)!}=
          (-1)^{n-k}binom{-n-1}{n-k}$$



          and the Vandermonde's identity.






          share|cite|improve this answer











          $endgroup$













          • $begingroup$
            I am feeling so dumb right now! Thanks!
            $endgroup$
            – samjoe
            Jun 25 '17 at 15:17










          • $begingroup$
            @Robert, could you explain how you went from expression after first '=' to the one after the second '='? Thanks
            $endgroup$
            – sku
            Jul 4 '17 at 23:20










          • $begingroup$
            @sku Yes. See my edited answer.
            $endgroup$
            – Robert Z
            Jul 5 '17 at 4:17



















          1












          $begingroup$

          Another way is to exploit the Melzak's identity: $$fleft(x+yright)=xdbinom{x+n}{n}sum_{k=0}^{n}left(-1right)^{k}dbinom{n}{k}frac{fleft(y-kright)}{x+k},, x,yinmathbb{R},, xneq-k $$ where $f $ is an algebraic polynomial up to degree $n $. So taking $fleft(zright)=dbinom{z+n}{n}z$ we get $$sum_{k=0}^{n}left(-1right)^{k}dbinom{n}{k}dbinom{y+n-k}{n}frac{y-k}{-x-k}=-frac{dbinom{n+y+x}{n}}{xdbinom{x+n}{n}}left(x+yright)$$ so taking $y=n$ and the limit $xrightarrow-n$ we get $$sum_{k=0}^{n}left(-1right)^{k}dbinom{n}{k}dbinom{2n-k}{n}=-lim_{xrightarrow-n}frac{dbinom{2n+x}{n}}{xdbinom{x+n}{n}}left(n+xright)=color{red}{1}$$ as wanted.






          share|cite|improve this answer









          $endgroup$





















            1












            $begingroup$

            I think that the simpler and shorter way would be:
            $$
            eqalign{
            & sumlimits_{0, le ,k, le ,n} {left( { - 1} right)^{,k} left( matrix{
            n cr
            k cr} right)left( matrix{
            2n - k cr
            n cr} right)} = cr
            & = sumlimits_{0, le ,k, le ,n} {left( matrix{
            k - n - 1 cr
            k cr} right)left( matrix{
            2n - k cr
            n cr} right)} = cr
            & = sumlimits_{0, le ,k, le ,n} {left( matrix{
            k - n - 1 cr
            k cr} right)left( matrix{
            2n - k cr
            n - k cr} right)} = cr
            & = left( matrix{
            n cr
            n cr} right) = 1 cr}
            $$
            where




            • 1st step : Upper Negation $ left({ - 1} right)^{,k} left( matrix{
              n cr
              k cr}right)=left( matrix{
              {k-n-1} cr
              k cr}right)$

            • 2nd step: Symmetry $left( matrix{
              n cr
              k cr}right)=left( matrix{
              n cr
              {n-k} cr}right)quad |0 le text{integer} ,n$

            • 3rd step: Double Convolution
              $$
              sumlimits_{a, le ,k, le ,b} {left( matrix{
              k - c cr
              k - a cr} right)left( matrix{
              d - k cr
              b - k cr} right)} = left( matrix{
              d - c + 1 cr
              b - a cr} right)
              $$






            share|cite|improve this answer











            $endgroup$





















              0












              $begingroup$

              There is also a combinatorial solution. Consider the following problem:




              How many subsets of ${1,2,dots,2n}$ of size $n$ contain none of the numbers in ${1,2,dots,n}$?




              On the one hand, the answer is obviously $1$; the only such subset is ${n+1,n+2,dots, 2n}$.



              On the other hand, we can solve this with the principle of inclusion exclusion. Letting $E_i$ be the number of subsets of size $n$ which do contain $i$, we need to count $E_1^complementcap E_2^complementcap dots cap E_n^complement$. For any $k$ indices $i_1,dots,i_k$, the size of the intersection $E_{i_1}cap E_{i_2}cap dots cap E_{i_k}$ is $binom{2n-k}n$. Taking the alternating sum over all such subsets of indices, we get your sum.






              share|cite|improve this answer









              $endgroup$













                Your Answer





                StackExchange.ifUsing("editor", function () {
                return StackExchange.using("mathjaxEditing", function () {
                StackExchange.MarkdownEditor.creationCallbacks.add(function (editor, postfix) {
                StackExchange.mathjaxEditing.prepareWmdForMathJax(editor, postfix, [["$", "$"], ["\\(","\\)"]]);
                });
                });
                }, "mathjax-editing");

                StackExchange.ready(function() {
                var channelOptions = {
                tags: "".split(" "),
                id: "69"
                };
                initTagRenderer("".split(" "), "".split(" "), channelOptions);

                StackExchange.using("externalEditor", function() {
                // Have to fire editor after snippets, if snippets enabled
                if (StackExchange.settings.snippets.snippetsEnabled) {
                StackExchange.using("snippets", function() {
                createEditor();
                });
                }
                else {
                createEditor();
                }
                });

                function createEditor() {
                StackExchange.prepareEditor({
                heartbeatType: 'answer',
                autoActivateHeartbeat: false,
                convertImagesToLinks: true,
                noModals: true,
                showLowRepImageUploadWarning: true,
                reputationToPostImages: 10,
                bindNavPrevention: true,
                postfix: "",
                imageUploader: {
                brandingHtml: "Powered by u003ca class="icon-imgur-white" href="https://imgur.com/"u003eu003c/au003e",
                contentPolicyHtml: "User contributions licensed under u003ca href="https://creativecommons.org/licenses/by-sa/3.0/"u003ecc by-sa 3.0 with attribution requiredu003c/au003e u003ca href="https://stackoverflow.com/legal/content-policy"u003e(content policy)u003c/au003e",
                allowUrls: true
                },
                noCode: true, onDemand: true,
                discardSelector: ".discard-answer"
                ,immediatelyShowMarkdownHelp:true
                });


                }
                });














                draft saved

                draft discarded


















                StackExchange.ready(
                function () {
                StackExchange.openid.initPostLogin('.new-post-login', 'https%3a%2f%2fmath.stackexchange.com%2fquestions%2f2335811%2ffind-the-sum-of-series-sum-k-0n-1k-binomnk-binom2n-kn%23new-answer', 'question_page');
                }
                );

                Post as a guest















                Required, but never shown

























                5 Answers
                5






                active

                oldest

                votes








                5 Answers
                5






                active

                oldest

                votes









                active

                oldest

                votes






                active

                oldest

                votes









                2












                $begingroup$

                $newcommand{bbx}[1]{,bbox[15px,border:1px groove navy]{displaystyle{#1}},}
                newcommand{braces}[1]{leftlbrace,{#1},rightrbrace}
                newcommand{bracks}[1]{leftlbrack,{#1},rightrbrack}
                newcommand{dd}{mathrm{d}}
                newcommand{ds}[1]{displaystyle{#1}}
                newcommand{expo}[1]{,mathrm{e}^{#1},}
                newcommand{ic}{mathrm{i}}
                newcommand{mc}[1]{mathcal{#1}}
                newcommand{mrm}[1]{mathrm{#1}}
                newcommand{pars}[1]{left(,{#1},right)}
                newcommand{partiald}[3]{frac{partial^{#1} #2}{partial #3^{#1}}}
                newcommand{root}[2]{,sqrt[#1]{,{#2},},}
                newcommand{totald}[3]{frac{mathrm{d}^{#1} #2}{mathrm{d} #3^{#1}}}
                newcommand{verts}[1]{leftvert,{#1},rightvert}$
                begin{align}
                sum_{k = 0}^{n}pars{-1}^{k}{n choose k}{2n - k choose n} & =
                sum_{k = 0}^{n}pars{-1}^{k}{n choose k}bracks{z^{n}}pars{1 + z}^{2n - k}
                \[5mm] & =
                bracks{z^{n}}pars{1 + z}^{2n}
                sum_{k = 0}^{n}{n choose k}pars{-,{1 over 1 + z}}^{k}
                \[5mm] & =
                bracks{z^{n}}pars{1 + z}^{2n}pars{1 - {1 over 1 + z}}^{n} =
                bracks{z^{n}}pars{1 + z}^{n},z^{n} = bbx{large 1}
                end{align}






                share|cite|improve this answer









                $endgroup$













                • $begingroup$
                  Thanks for your answer! I think I need to practice more on algebraic manipulation.
                  $endgroup$
                  – samjoe
                  Jun 26 '17 at 3:31










                • $begingroup$
                  That's a good idea. Thanks. $left(angle quad angle atop {Huge smile}right)$
                  $endgroup$
                  – Felix Marin
                  Jun 26 '17 at 3:34










                • $begingroup$
                  Sorry for off-topic question - do you type out latex or use some software?
                  $endgroup$
                  – samjoe
                  Jun 26 '17 at 3:45










                • $begingroup$
                  I type $LaTeX$. The above icon is $LaTeX$ too. It's the code: $texttt{$left(anglequadangleatop{Hugesmile}right)$}$
                  $endgroup$
                  – Felix Marin
                  Jun 26 '17 at 3:59


















                2












                $begingroup$

                $newcommand{bbx}[1]{,bbox[15px,border:1px groove navy]{displaystyle{#1}},}
                newcommand{braces}[1]{leftlbrace,{#1},rightrbrace}
                newcommand{bracks}[1]{leftlbrack,{#1},rightrbrack}
                newcommand{dd}{mathrm{d}}
                newcommand{ds}[1]{displaystyle{#1}}
                newcommand{expo}[1]{,mathrm{e}^{#1},}
                newcommand{ic}{mathrm{i}}
                newcommand{mc}[1]{mathcal{#1}}
                newcommand{mrm}[1]{mathrm{#1}}
                newcommand{pars}[1]{left(,{#1},right)}
                newcommand{partiald}[3]{frac{partial^{#1} #2}{partial #3^{#1}}}
                newcommand{root}[2]{,sqrt[#1]{,{#2},},}
                newcommand{totald}[3]{frac{mathrm{d}^{#1} #2}{mathrm{d} #3^{#1}}}
                newcommand{verts}[1]{leftvert,{#1},rightvert}$
                begin{align}
                sum_{k = 0}^{n}pars{-1}^{k}{n choose k}{2n - k choose n} & =
                sum_{k = 0}^{n}pars{-1}^{k}{n choose k}bracks{z^{n}}pars{1 + z}^{2n - k}
                \[5mm] & =
                bracks{z^{n}}pars{1 + z}^{2n}
                sum_{k = 0}^{n}{n choose k}pars{-,{1 over 1 + z}}^{k}
                \[5mm] & =
                bracks{z^{n}}pars{1 + z}^{2n}pars{1 - {1 over 1 + z}}^{n} =
                bracks{z^{n}}pars{1 + z}^{n},z^{n} = bbx{large 1}
                end{align}






                share|cite|improve this answer









                $endgroup$













                • $begingroup$
                  Thanks for your answer! I think I need to practice more on algebraic manipulation.
                  $endgroup$
                  – samjoe
                  Jun 26 '17 at 3:31










                • $begingroup$
                  That's a good idea. Thanks. $left(angle quad angle atop {Huge smile}right)$
                  $endgroup$
                  – Felix Marin
                  Jun 26 '17 at 3:34










                • $begingroup$
                  Sorry for off-topic question - do you type out latex or use some software?
                  $endgroup$
                  – samjoe
                  Jun 26 '17 at 3:45










                • $begingroup$
                  I type $LaTeX$. The above icon is $LaTeX$ too. It's the code: $texttt{$left(anglequadangleatop{Hugesmile}right)$}$
                  $endgroup$
                  – Felix Marin
                  Jun 26 '17 at 3:59
















                2












                2








                2





                $begingroup$

                $newcommand{bbx}[1]{,bbox[15px,border:1px groove navy]{displaystyle{#1}},}
                newcommand{braces}[1]{leftlbrace,{#1},rightrbrace}
                newcommand{bracks}[1]{leftlbrack,{#1},rightrbrack}
                newcommand{dd}{mathrm{d}}
                newcommand{ds}[1]{displaystyle{#1}}
                newcommand{expo}[1]{,mathrm{e}^{#1},}
                newcommand{ic}{mathrm{i}}
                newcommand{mc}[1]{mathcal{#1}}
                newcommand{mrm}[1]{mathrm{#1}}
                newcommand{pars}[1]{left(,{#1},right)}
                newcommand{partiald}[3]{frac{partial^{#1} #2}{partial #3^{#1}}}
                newcommand{root}[2]{,sqrt[#1]{,{#2},},}
                newcommand{totald}[3]{frac{mathrm{d}^{#1} #2}{mathrm{d} #3^{#1}}}
                newcommand{verts}[1]{leftvert,{#1},rightvert}$
                begin{align}
                sum_{k = 0}^{n}pars{-1}^{k}{n choose k}{2n - k choose n} & =
                sum_{k = 0}^{n}pars{-1}^{k}{n choose k}bracks{z^{n}}pars{1 + z}^{2n - k}
                \[5mm] & =
                bracks{z^{n}}pars{1 + z}^{2n}
                sum_{k = 0}^{n}{n choose k}pars{-,{1 over 1 + z}}^{k}
                \[5mm] & =
                bracks{z^{n}}pars{1 + z}^{2n}pars{1 - {1 over 1 + z}}^{n} =
                bracks{z^{n}}pars{1 + z}^{n},z^{n} = bbx{large 1}
                end{align}






                share|cite|improve this answer









                $endgroup$



                $newcommand{bbx}[1]{,bbox[15px,border:1px groove navy]{displaystyle{#1}},}
                newcommand{braces}[1]{leftlbrace,{#1},rightrbrace}
                newcommand{bracks}[1]{leftlbrack,{#1},rightrbrack}
                newcommand{dd}{mathrm{d}}
                newcommand{ds}[1]{displaystyle{#1}}
                newcommand{expo}[1]{,mathrm{e}^{#1},}
                newcommand{ic}{mathrm{i}}
                newcommand{mc}[1]{mathcal{#1}}
                newcommand{mrm}[1]{mathrm{#1}}
                newcommand{pars}[1]{left(,{#1},right)}
                newcommand{partiald}[3]{frac{partial^{#1} #2}{partial #3^{#1}}}
                newcommand{root}[2]{,sqrt[#1]{,{#2},},}
                newcommand{totald}[3]{frac{mathrm{d}^{#1} #2}{mathrm{d} #3^{#1}}}
                newcommand{verts}[1]{leftvert,{#1},rightvert}$
                begin{align}
                sum_{k = 0}^{n}pars{-1}^{k}{n choose k}{2n - k choose n} & =
                sum_{k = 0}^{n}pars{-1}^{k}{n choose k}bracks{z^{n}}pars{1 + z}^{2n - k}
                \[5mm] & =
                bracks{z^{n}}pars{1 + z}^{2n}
                sum_{k = 0}^{n}{n choose k}pars{-,{1 over 1 + z}}^{k}
                \[5mm] & =
                bracks{z^{n}}pars{1 + z}^{2n}pars{1 - {1 over 1 + z}}^{n} =
                bracks{z^{n}}pars{1 + z}^{n},z^{n} = bbx{large 1}
                end{align}







                share|cite|improve this answer












                share|cite|improve this answer



                share|cite|improve this answer










                answered Jun 25 '17 at 19:14









                Felix MarinFelix Marin

                68.6k7109145




                68.6k7109145












                • $begingroup$
                  Thanks for your answer! I think I need to practice more on algebraic manipulation.
                  $endgroup$
                  – samjoe
                  Jun 26 '17 at 3:31










                • $begingroup$
                  That's a good idea. Thanks. $left(angle quad angle atop {Huge smile}right)$
                  $endgroup$
                  – Felix Marin
                  Jun 26 '17 at 3:34










                • $begingroup$
                  Sorry for off-topic question - do you type out latex or use some software?
                  $endgroup$
                  – samjoe
                  Jun 26 '17 at 3:45










                • $begingroup$
                  I type $LaTeX$. The above icon is $LaTeX$ too. It's the code: $texttt{$left(anglequadangleatop{Hugesmile}right)$}$
                  $endgroup$
                  – Felix Marin
                  Jun 26 '17 at 3:59




















                • $begingroup$
                  Thanks for your answer! I think I need to practice more on algebraic manipulation.
                  $endgroup$
                  – samjoe
                  Jun 26 '17 at 3:31










                • $begingroup$
                  That's a good idea. Thanks. $left(angle quad angle atop {Huge smile}right)$
                  $endgroup$
                  – Felix Marin
                  Jun 26 '17 at 3:34










                • $begingroup$
                  Sorry for off-topic question - do you type out latex or use some software?
                  $endgroup$
                  – samjoe
                  Jun 26 '17 at 3:45










                • $begingroup$
                  I type $LaTeX$. The above icon is $LaTeX$ too. It's the code: $texttt{$left(anglequadangleatop{Hugesmile}right)$}$
                  $endgroup$
                  – Felix Marin
                  Jun 26 '17 at 3:59


















                $begingroup$
                Thanks for your answer! I think I need to practice more on algebraic manipulation.
                $endgroup$
                – samjoe
                Jun 26 '17 at 3:31




                $begingroup$
                Thanks for your answer! I think I need to practice more on algebraic manipulation.
                $endgroup$
                – samjoe
                Jun 26 '17 at 3:31












                $begingroup$
                That's a good idea. Thanks. $left(angle quad angle atop {Huge smile}right)$
                $endgroup$
                – Felix Marin
                Jun 26 '17 at 3:34




                $begingroup$
                That's a good idea. Thanks. $left(angle quad angle atop {Huge smile}right)$
                $endgroup$
                – Felix Marin
                Jun 26 '17 at 3:34












                $begingroup$
                Sorry for off-topic question - do you type out latex or use some software?
                $endgroup$
                – samjoe
                Jun 26 '17 at 3:45




                $begingroup$
                Sorry for off-topic question - do you type out latex or use some software?
                $endgroup$
                – samjoe
                Jun 26 '17 at 3:45












                $begingroup$
                I type $LaTeX$. The above icon is $LaTeX$ too. It's the code: $texttt{$left(anglequadangleatop{Hugesmile}right)$}$
                $endgroup$
                – Felix Marin
                Jun 26 '17 at 3:59






                $begingroup$
                I type $LaTeX$. The above icon is $LaTeX$ too. It's the code: $texttt{$left(anglequadangleatop{Hugesmile}right)$}$
                $endgroup$
                – Felix Marin
                Jun 26 '17 at 3:59













                2












                $begingroup$

                Note that
                begin{align*}
                sum_{k=0}^{n} (-1)^k binom{n}{k} binom{2n-k}{n}&=sum_{k=0}^{n} (-1)^k binom{n}{k} binom{2n-k}{n-k}\
                &=sum_{k=0}^{n} (-1)^k binom{n}{k} (-1)^{n-k}binom{-(n+1)}{n-k}\
                &=(-1)^{n}sum_{k=0}^{n} binom{n}{k} binom{-(n+1)}{n-k}\
                &=(-1)^{n}binom{n-(n+1)}{n}=1.
                end{align*}
                where we used
                $$binom{2n-k}{n-k}=frac{(2n-k)cdots(n+1)}{(n-k)!}=
                (-1)^{n-k}frac{(-n-1)cdots(-2n+k)}{(n-k)!}=
                (-1)^{n-k}binom{-n-1}{n-k}$$



                and the Vandermonde's identity.






                share|cite|improve this answer











                $endgroup$













                • $begingroup$
                  I am feeling so dumb right now! Thanks!
                  $endgroup$
                  – samjoe
                  Jun 25 '17 at 15:17










                • $begingroup$
                  @Robert, could you explain how you went from expression after first '=' to the one after the second '='? Thanks
                  $endgroup$
                  – sku
                  Jul 4 '17 at 23:20










                • $begingroup$
                  @sku Yes. See my edited answer.
                  $endgroup$
                  – Robert Z
                  Jul 5 '17 at 4:17
















                2












                $begingroup$

                Note that
                begin{align*}
                sum_{k=0}^{n} (-1)^k binom{n}{k} binom{2n-k}{n}&=sum_{k=0}^{n} (-1)^k binom{n}{k} binom{2n-k}{n-k}\
                &=sum_{k=0}^{n} (-1)^k binom{n}{k} (-1)^{n-k}binom{-(n+1)}{n-k}\
                &=(-1)^{n}sum_{k=0}^{n} binom{n}{k} binom{-(n+1)}{n-k}\
                &=(-1)^{n}binom{n-(n+1)}{n}=1.
                end{align*}
                where we used
                $$binom{2n-k}{n-k}=frac{(2n-k)cdots(n+1)}{(n-k)!}=
                (-1)^{n-k}frac{(-n-1)cdots(-2n+k)}{(n-k)!}=
                (-1)^{n-k}binom{-n-1}{n-k}$$



                and the Vandermonde's identity.






                share|cite|improve this answer











                $endgroup$













                • $begingroup$
                  I am feeling so dumb right now! Thanks!
                  $endgroup$
                  – samjoe
                  Jun 25 '17 at 15:17










                • $begingroup$
                  @Robert, could you explain how you went from expression after first '=' to the one after the second '='? Thanks
                  $endgroup$
                  – sku
                  Jul 4 '17 at 23:20










                • $begingroup$
                  @sku Yes. See my edited answer.
                  $endgroup$
                  – Robert Z
                  Jul 5 '17 at 4:17














                2












                2








                2





                $begingroup$

                Note that
                begin{align*}
                sum_{k=0}^{n} (-1)^k binom{n}{k} binom{2n-k}{n}&=sum_{k=0}^{n} (-1)^k binom{n}{k} binom{2n-k}{n-k}\
                &=sum_{k=0}^{n} (-1)^k binom{n}{k} (-1)^{n-k}binom{-(n+1)}{n-k}\
                &=(-1)^{n}sum_{k=0}^{n} binom{n}{k} binom{-(n+1)}{n-k}\
                &=(-1)^{n}binom{n-(n+1)}{n}=1.
                end{align*}
                where we used
                $$binom{2n-k}{n-k}=frac{(2n-k)cdots(n+1)}{(n-k)!}=
                (-1)^{n-k}frac{(-n-1)cdots(-2n+k)}{(n-k)!}=
                (-1)^{n-k}binom{-n-1}{n-k}$$



                and the Vandermonde's identity.






                share|cite|improve this answer











                $endgroup$



                Note that
                begin{align*}
                sum_{k=0}^{n} (-1)^k binom{n}{k} binom{2n-k}{n}&=sum_{k=0}^{n} (-1)^k binom{n}{k} binom{2n-k}{n-k}\
                &=sum_{k=0}^{n} (-1)^k binom{n}{k} (-1)^{n-k}binom{-(n+1)}{n-k}\
                &=(-1)^{n}sum_{k=0}^{n} binom{n}{k} binom{-(n+1)}{n-k}\
                &=(-1)^{n}binom{n-(n+1)}{n}=1.
                end{align*}
                where we used
                $$binom{2n-k}{n-k}=frac{(2n-k)cdots(n+1)}{(n-k)!}=
                (-1)^{n-k}frac{(-n-1)cdots(-2n+k)}{(n-k)!}=
                (-1)^{n-k}binom{-n-1}{n-k}$$



                and the Vandermonde's identity.







                share|cite|improve this answer














                share|cite|improve this answer



                share|cite|improve this answer








                edited Jul 5 '17 at 4:16

























                answered Jun 25 '17 at 15:16









                Robert ZRobert Z

                101k1069142




                101k1069142












                • $begingroup$
                  I am feeling so dumb right now! Thanks!
                  $endgroup$
                  – samjoe
                  Jun 25 '17 at 15:17










                • $begingroup$
                  @Robert, could you explain how you went from expression after first '=' to the one after the second '='? Thanks
                  $endgroup$
                  – sku
                  Jul 4 '17 at 23:20










                • $begingroup$
                  @sku Yes. See my edited answer.
                  $endgroup$
                  – Robert Z
                  Jul 5 '17 at 4:17


















                • $begingroup$
                  I am feeling so dumb right now! Thanks!
                  $endgroup$
                  – samjoe
                  Jun 25 '17 at 15:17










                • $begingroup$
                  @Robert, could you explain how you went from expression after first '=' to the one after the second '='? Thanks
                  $endgroup$
                  – sku
                  Jul 4 '17 at 23:20










                • $begingroup$
                  @sku Yes. See my edited answer.
                  $endgroup$
                  – Robert Z
                  Jul 5 '17 at 4:17
















                $begingroup$
                I am feeling so dumb right now! Thanks!
                $endgroup$
                – samjoe
                Jun 25 '17 at 15:17




                $begingroup$
                I am feeling so dumb right now! Thanks!
                $endgroup$
                – samjoe
                Jun 25 '17 at 15:17












                $begingroup$
                @Robert, could you explain how you went from expression after first '=' to the one after the second '='? Thanks
                $endgroup$
                – sku
                Jul 4 '17 at 23:20




                $begingroup$
                @Robert, could you explain how you went from expression after first '=' to the one after the second '='? Thanks
                $endgroup$
                – sku
                Jul 4 '17 at 23:20












                $begingroup$
                @sku Yes. See my edited answer.
                $endgroup$
                – Robert Z
                Jul 5 '17 at 4:17




                $begingroup$
                @sku Yes. See my edited answer.
                $endgroup$
                – Robert Z
                Jul 5 '17 at 4:17











                1












                $begingroup$

                Another way is to exploit the Melzak's identity: $$fleft(x+yright)=xdbinom{x+n}{n}sum_{k=0}^{n}left(-1right)^{k}dbinom{n}{k}frac{fleft(y-kright)}{x+k},, x,yinmathbb{R},, xneq-k $$ where $f $ is an algebraic polynomial up to degree $n $. So taking $fleft(zright)=dbinom{z+n}{n}z$ we get $$sum_{k=0}^{n}left(-1right)^{k}dbinom{n}{k}dbinom{y+n-k}{n}frac{y-k}{-x-k}=-frac{dbinom{n+y+x}{n}}{xdbinom{x+n}{n}}left(x+yright)$$ so taking $y=n$ and the limit $xrightarrow-n$ we get $$sum_{k=0}^{n}left(-1right)^{k}dbinom{n}{k}dbinom{2n-k}{n}=-lim_{xrightarrow-n}frac{dbinom{2n+x}{n}}{xdbinom{x+n}{n}}left(n+xright)=color{red}{1}$$ as wanted.






                share|cite|improve this answer









                $endgroup$


















                  1












                  $begingroup$

                  Another way is to exploit the Melzak's identity: $$fleft(x+yright)=xdbinom{x+n}{n}sum_{k=0}^{n}left(-1right)^{k}dbinom{n}{k}frac{fleft(y-kright)}{x+k},, x,yinmathbb{R},, xneq-k $$ where $f $ is an algebraic polynomial up to degree $n $. So taking $fleft(zright)=dbinom{z+n}{n}z$ we get $$sum_{k=0}^{n}left(-1right)^{k}dbinom{n}{k}dbinom{y+n-k}{n}frac{y-k}{-x-k}=-frac{dbinom{n+y+x}{n}}{xdbinom{x+n}{n}}left(x+yright)$$ so taking $y=n$ and the limit $xrightarrow-n$ we get $$sum_{k=0}^{n}left(-1right)^{k}dbinom{n}{k}dbinom{2n-k}{n}=-lim_{xrightarrow-n}frac{dbinom{2n+x}{n}}{xdbinom{x+n}{n}}left(n+xright)=color{red}{1}$$ as wanted.






                  share|cite|improve this answer









                  $endgroup$
















                    1












                    1








                    1





                    $begingroup$

                    Another way is to exploit the Melzak's identity: $$fleft(x+yright)=xdbinom{x+n}{n}sum_{k=0}^{n}left(-1right)^{k}dbinom{n}{k}frac{fleft(y-kright)}{x+k},, x,yinmathbb{R},, xneq-k $$ where $f $ is an algebraic polynomial up to degree $n $. So taking $fleft(zright)=dbinom{z+n}{n}z$ we get $$sum_{k=0}^{n}left(-1right)^{k}dbinom{n}{k}dbinom{y+n-k}{n}frac{y-k}{-x-k}=-frac{dbinom{n+y+x}{n}}{xdbinom{x+n}{n}}left(x+yright)$$ so taking $y=n$ and the limit $xrightarrow-n$ we get $$sum_{k=0}^{n}left(-1right)^{k}dbinom{n}{k}dbinom{2n-k}{n}=-lim_{xrightarrow-n}frac{dbinom{2n+x}{n}}{xdbinom{x+n}{n}}left(n+xright)=color{red}{1}$$ as wanted.






                    share|cite|improve this answer









                    $endgroup$



                    Another way is to exploit the Melzak's identity: $$fleft(x+yright)=xdbinom{x+n}{n}sum_{k=0}^{n}left(-1right)^{k}dbinom{n}{k}frac{fleft(y-kright)}{x+k},, x,yinmathbb{R},, xneq-k $$ where $f $ is an algebraic polynomial up to degree $n $. So taking $fleft(zright)=dbinom{z+n}{n}z$ we get $$sum_{k=0}^{n}left(-1right)^{k}dbinom{n}{k}dbinom{y+n-k}{n}frac{y-k}{-x-k}=-frac{dbinom{n+y+x}{n}}{xdbinom{x+n}{n}}left(x+yright)$$ so taking $y=n$ and the limit $xrightarrow-n$ we get $$sum_{k=0}^{n}left(-1right)^{k}dbinom{n}{k}dbinom{2n-k}{n}=-lim_{xrightarrow-n}frac{dbinom{2n+x}{n}}{xdbinom{x+n}{n}}left(n+xright)=color{red}{1}$$ as wanted.







                    share|cite|improve this answer












                    share|cite|improve this answer



                    share|cite|improve this answer










                    answered Jul 6 '17 at 6:53









                    Marco CantariniMarco Cantarini

                    29.4k23373




                    29.4k23373























                        1












                        $begingroup$

                        I think that the simpler and shorter way would be:
                        $$
                        eqalign{
                        & sumlimits_{0, le ,k, le ,n} {left( { - 1} right)^{,k} left( matrix{
                        n cr
                        k cr} right)left( matrix{
                        2n - k cr
                        n cr} right)} = cr
                        & = sumlimits_{0, le ,k, le ,n} {left( matrix{
                        k - n - 1 cr
                        k cr} right)left( matrix{
                        2n - k cr
                        n cr} right)} = cr
                        & = sumlimits_{0, le ,k, le ,n} {left( matrix{
                        k - n - 1 cr
                        k cr} right)left( matrix{
                        2n - k cr
                        n - k cr} right)} = cr
                        & = left( matrix{
                        n cr
                        n cr} right) = 1 cr}
                        $$
                        where




                        • 1st step : Upper Negation $ left({ - 1} right)^{,k} left( matrix{
                          n cr
                          k cr}right)=left( matrix{
                          {k-n-1} cr
                          k cr}right)$

                        • 2nd step: Symmetry $left( matrix{
                          n cr
                          k cr}right)=left( matrix{
                          n cr
                          {n-k} cr}right)quad |0 le text{integer} ,n$

                        • 3rd step: Double Convolution
                          $$
                          sumlimits_{a, le ,k, le ,b} {left( matrix{
                          k - c cr
                          k - a cr} right)left( matrix{
                          d - k cr
                          b - k cr} right)} = left( matrix{
                          d - c + 1 cr
                          b - a cr} right)
                          $$






                        share|cite|improve this answer











                        $endgroup$


















                          1












                          $begingroup$

                          I think that the simpler and shorter way would be:
                          $$
                          eqalign{
                          & sumlimits_{0, le ,k, le ,n} {left( { - 1} right)^{,k} left( matrix{
                          n cr
                          k cr} right)left( matrix{
                          2n - k cr
                          n cr} right)} = cr
                          & = sumlimits_{0, le ,k, le ,n} {left( matrix{
                          k - n - 1 cr
                          k cr} right)left( matrix{
                          2n - k cr
                          n cr} right)} = cr
                          & = sumlimits_{0, le ,k, le ,n} {left( matrix{
                          k - n - 1 cr
                          k cr} right)left( matrix{
                          2n - k cr
                          n - k cr} right)} = cr
                          & = left( matrix{
                          n cr
                          n cr} right) = 1 cr}
                          $$
                          where




                          • 1st step : Upper Negation $ left({ - 1} right)^{,k} left( matrix{
                            n cr
                            k cr}right)=left( matrix{
                            {k-n-1} cr
                            k cr}right)$

                          • 2nd step: Symmetry $left( matrix{
                            n cr
                            k cr}right)=left( matrix{
                            n cr
                            {n-k} cr}right)quad |0 le text{integer} ,n$

                          • 3rd step: Double Convolution
                            $$
                            sumlimits_{a, le ,k, le ,b} {left( matrix{
                            k - c cr
                            k - a cr} right)left( matrix{
                            d - k cr
                            b - k cr} right)} = left( matrix{
                            d - c + 1 cr
                            b - a cr} right)
                            $$






                          share|cite|improve this answer











                          $endgroup$
















                            1












                            1








                            1





                            $begingroup$

                            I think that the simpler and shorter way would be:
                            $$
                            eqalign{
                            & sumlimits_{0, le ,k, le ,n} {left( { - 1} right)^{,k} left( matrix{
                            n cr
                            k cr} right)left( matrix{
                            2n - k cr
                            n cr} right)} = cr
                            & = sumlimits_{0, le ,k, le ,n} {left( matrix{
                            k - n - 1 cr
                            k cr} right)left( matrix{
                            2n - k cr
                            n cr} right)} = cr
                            & = sumlimits_{0, le ,k, le ,n} {left( matrix{
                            k - n - 1 cr
                            k cr} right)left( matrix{
                            2n - k cr
                            n - k cr} right)} = cr
                            & = left( matrix{
                            n cr
                            n cr} right) = 1 cr}
                            $$
                            where




                            • 1st step : Upper Negation $ left({ - 1} right)^{,k} left( matrix{
                              n cr
                              k cr}right)=left( matrix{
                              {k-n-1} cr
                              k cr}right)$

                            • 2nd step: Symmetry $left( matrix{
                              n cr
                              k cr}right)=left( matrix{
                              n cr
                              {n-k} cr}right)quad |0 le text{integer} ,n$

                            • 3rd step: Double Convolution
                              $$
                              sumlimits_{a, le ,k, le ,b} {left( matrix{
                              k - c cr
                              k - a cr} right)left( matrix{
                              d - k cr
                              b - k cr} right)} = left( matrix{
                              d - c + 1 cr
                              b - a cr} right)
                              $$






                            share|cite|improve this answer











                            $endgroup$



                            I think that the simpler and shorter way would be:
                            $$
                            eqalign{
                            & sumlimits_{0, le ,k, le ,n} {left( { - 1} right)^{,k} left( matrix{
                            n cr
                            k cr} right)left( matrix{
                            2n - k cr
                            n cr} right)} = cr
                            & = sumlimits_{0, le ,k, le ,n} {left( matrix{
                            k - n - 1 cr
                            k cr} right)left( matrix{
                            2n - k cr
                            n cr} right)} = cr
                            & = sumlimits_{0, le ,k, le ,n} {left( matrix{
                            k - n - 1 cr
                            k cr} right)left( matrix{
                            2n - k cr
                            n - k cr} right)} = cr
                            & = left( matrix{
                            n cr
                            n cr} right) = 1 cr}
                            $$
                            where




                            • 1st step : Upper Negation $ left({ - 1} right)^{,k} left( matrix{
                              n cr
                              k cr}right)=left( matrix{
                              {k-n-1} cr
                              k cr}right)$

                            • 2nd step: Symmetry $left( matrix{
                              n cr
                              k cr}right)=left( matrix{
                              n cr
                              {n-k} cr}right)quad |0 le text{integer} ,n$

                            • 3rd step: Double Convolution
                              $$
                              sumlimits_{a, le ,k, le ,b} {left( matrix{
                              k - c cr
                              k - a cr} right)left( matrix{
                              d - k cr
                              b - k cr} right)} = left( matrix{
                              d - c + 1 cr
                              b - a cr} right)
                              $$







                            share|cite|improve this answer














                            share|cite|improve this answer



                            share|cite|improve this answer








                            edited Jul 8 '17 at 15:14

























                            answered Jul 6 '17 at 23:23









                            G CabG Cab

                            20.1k31340




                            20.1k31340























                                0












                                $begingroup$

                                There is also a combinatorial solution. Consider the following problem:




                                How many subsets of ${1,2,dots,2n}$ of size $n$ contain none of the numbers in ${1,2,dots,n}$?




                                On the one hand, the answer is obviously $1$; the only such subset is ${n+1,n+2,dots, 2n}$.



                                On the other hand, we can solve this with the principle of inclusion exclusion. Letting $E_i$ be the number of subsets of size $n$ which do contain $i$, we need to count $E_1^complementcap E_2^complementcap dots cap E_n^complement$. For any $k$ indices $i_1,dots,i_k$, the size of the intersection $E_{i_1}cap E_{i_2}cap dots cap E_{i_k}$ is $binom{2n-k}n$. Taking the alternating sum over all such subsets of indices, we get your sum.






                                share|cite|improve this answer









                                $endgroup$


















                                  0












                                  $begingroup$

                                  There is also a combinatorial solution. Consider the following problem:




                                  How many subsets of ${1,2,dots,2n}$ of size $n$ contain none of the numbers in ${1,2,dots,n}$?




                                  On the one hand, the answer is obviously $1$; the only such subset is ${n+1,n+2,dots, 2n}$.



                                  On the other hand, we can solve this with the principle of inclusion exclusion. Letting $E_i$ be the number of subsets of size $n$ which do contain $i$, we need to count $E_1^complementcap E_2^complementcap dots cap E_n^complement$. For any $k$ indices $i_1,dots,i_k$, the size of the intersection $E_{i_1}cap E_{i_2}cap dots cap E_{i_k}$ is $binom{2n-k}n$. Taking the alternating sum over all such subsets of indices, we get your sum.






                                  share|cite|improve this answer









                                  $endgroup$
















                                    0












                                    0








                                    0





                                    $begingroup$

                                    There is also a combinatorial solution. Consider the following problem:




                                    How many subsets of ${1,2,dots,2n}$ of size $n$ contain none of the numbers in ${1,2,dots,n}$?




                                    On the one hand, the answer is obviously $1$; the only such subset is ${n+1,n+2,dots, 2n}$.



                                    On the other hand, we can solve this with the principle of inclusion exclusion. Letting $E_i$ be the number of subsets of size $n$ which do contain $i$, we need to count $E_1^complementcap E_2^complementcap dots cap E_n^complement$. For any $k$ indices $i_1,dots,i_k$, the size of the intersection $E_{i_1}cap E_{i_2}cap dots cap E_{i_k}$ is $binom{2n-k}n$. Taking the alternating sum over all such subsets of indices, we get your sum.






                                    share|cite|improve this answer









                                    $endgroup$



                                    There is also a combinatorial solution. Consider the following problem:




                                    How many subsets of ${1,2,dots,2n}$ of size $n$ contain none of the numbers in ${1,2,dots,n}$?




                                    On the one hand, the answer is obviously $1$; the only such subset is ${n+1,n+2,dots, 2n}$.



                                    On the other hand, we can solve this with the principle of inclusion exclusion. Letting $E_i$ be the number of subsets of size $n$ which do contain $i$, we need to count $E_1^complementcap E_2^complementcap dots cap E_n^complement$. For any $k$ indices $i_1,dots,i_k$, the size of the intersection $E_{i_1}cap E_{i_2}cap dots cap E_{i_k}$ is $binom{2n-k}n$. Taking the alternating sum over all such subsets of indices, we get your sum.







                                    share|cite|improve this answer












                                    share|cite|improve this answer



                                    share|cite|improve this answer










                                    answered Dec 27 '18 at 18:47









                                    Mike EarnestMike Earnest

                                    24.9k22151




                                    24.9k22151






























                                        draft saved

                                        draft discarded




















































                                        Thanks for contributing an answer to Mathematics Stack Exchange!


                                        • Please be sure to answer the question. Provide details and share your research!

                                        But avoid



                                        • Asking for help, clarification, or responding to other answers.

                                        • Making statements based on opinion; back them up with references or personal experience.


                                        Use MathJax to format equations. MathJax reference.


                                        To learn more, see our tips on writing great answers.




                                        draft saved


                                        draft discarded














                                        StackExchange.ready(
                                        function () {
                                        StackExchange.openid.initPostLogin('.new-post-login', 'https%3a%2f%2fmath.stackexchange.com%2fquestions%2f2335811%2ffind-the-sum-of-series-sum-k-0n-1k-binomnk-binom2n-kn%23new-answer', 'question_page');
                                        }
                                        );

                                        Post as a guest















                                        Required, but never shown





















































                                        Required, but never shown














                                        Required, but never shown












                                        Required, but never shown







                                        Required, but never shown

































                                        Required, but never shown














                                        Required, but never shown












                                        Required, but never shown







                                        Required, but never shown







                                        Popular posts from this blog

                                        To store a contact into the json file from server.js file using a class in NodeJS

                                        Redirect URL with Chrome Remote Debugging Android Devices

                                        Dieringhausen